Is there some approach to make functional integrals rigorous?












4












$begingroup$


Quantum Mechanics and Quantum Field Theory can both be formulated in terms of the so-called functional integrals.



The point is that intuitively it is an "integral over all possible paths" or rather "integral over all possible field configurations", traditionaly denoted as



$$int A[gamma(t)]mathcal{D}gamma(t)$$



$$int A[phi(x)]mathcal{D}phi(x)$$



for respectively paths and fields. It seems however that this is not well defined. I really don't understand how can one manipulate something that isn't defined, so I'm searching for the right way to understand these things.



Is there some way to make sense of these objects? I heard that as traditional measures it is not possible, but is there any other alternative way to make this be defined? If there is no way, how can someone work with one object that has no meaning associated with it and compute things with it?










share|cite|improve this question









$endgroup$












  • $begingroup$
    take a look here
    $endgroup$
    – Masacroso
    May 18 '17 at 0:24
















4












$begingroup$


Quantum Mechanics and Quantum Field Theory can both be formulated in terms of the so-called functional integrals.



The point is that intuitively it is an "integral over all possible paths" or rather "integral over all possible field configurations", traditionaly denoted as



$$int A[gamma(t)]mathcal{D}gamma(t)$$



$$int A[phi(x)]mathcal{D}phi(x)$$



for respectively paths and fields. It seems however that this is not well defined. I really don't understand how can one manipulate something that isn't defined, so I'm searching for the right way to understand these things.



Is there some way to make sense of these objects? I heard that as traditional measures it is not possible, but is there any other alternative way to make this be defined? If there is no way, how can someone work with one object that has no meaning associated with it and compute things with it?










share|cite|improve this question









$endgroup$












  • $begingroup$
    take a look here
    $endgroup$
    – Masacroso
    May 18 '17 at 0:24














4












4








4


2



$begingroup$


Quantum Mechanics and Quantum Field Theory can both be formulated in terms of the so-called functional integrals.



The point is that intuitively it is an "integral over all possible paths" or rather "integral over all possible field configurations", traditionaly denoted as



$$int A[gamma(t)]mathcal{D}gamma(t)$$



$$int A[phi(x)]mathcal{D}phi(x)$$



for respectively paths and fields. It seems however that this is not well defined. I really don't understand how can one manipulate something that isn't defined, so I'm searching for the right way to understand these things.



Is there some way to make sense of these objects? I heard that as traditional measures it is not possible, but is there any other alternative way to make this be defined? If there is no way, how can someone work with one object that has no meaning associated with it and compute things with it?










share|cite|improve this question









$endgroup$




Quantum Mechanics and Quantum Field Theory can both be formulated in terms of the so-called functional integrals.



The point is that intuitively it is an "integral over all possible paths" or rather "integral over all possible field configurations", traditionaly denoted as



$$int A[gamma(t)]mathcal{D}gamma(t)$$



$$int A[phi(x)]mathcal{D}phi(x)$$



for respectively paths and fields. It seems however that this is not well defined. I really don't understand how can one manipulate something that isn't defined, so I'm searching for the right way to understand these things.



Is there some way to make sense of these objects? I heard that as traditional measures it is not possible, but is there any other alternative way to make this be defined? If there is no way, how can someone work with one object that has no meaning associated with it and compute things with it?







functional-analysis measure-theory mathematical-physics quantum-field-theory






share|cite|improve this question













share|cite|improve this question











share|cite|improve this question




share|cite|improve this question










asked May 18 '17 at 0:14









user1620696user1620696

11.7k642116




11.7k642116












  • $begingroup$
    take a look here
    $endgroup$
    – Masacroso
    May 18 '17 at 0:24


















  • $begingroup$
    take a look here
    $endgroup$
    – Masacroso
    May 18 '17 at 0:24
















$begingroup$
take a look here
$endgroup$
– Masacroso
May 18 '17 at 0:24




$begingroup$
take a look here
$endgroup$
– Masacroso
May 18 '17 at 0:24










2 Answers
2






active

oldest

votes


















3












$begingroup$

There are two versions of such path integrals: the Minkowskian and the Euclidean.
The first one is of the form
$$
int F(phi) e^{iS(phi)} Dphi
$$

and the second one is
$$
int F(phi) e^{-S(phi)} Dphi .
$$

Here $S$ is the action functional and $F$ is another functional corresponding to observables.
The Minkowskian case lies outside ordinary measure theory, even in the Gaussian case (a Theorem by Cameron https://onlinelibrary.wiley.com/doi/abs/10.1002/sapm1960391126 correcting an earlier mistake by Gelfand and Yaglom who thought that $sigma$-additive complex measures would work). One instead has to use a limit of a time slicing procedure to make sense of it.



For the Euclidean case, ordinary measure theory is perfectly adequate.
The "I heard that as traditional measures it is not possible" is a common misconception that unfortunately gets repeated ad infinitum. The part $e^{-S(phi)} Dphi$ should be a Borel probability measure on a space of distributions like $mathscr{S}'(mathbb{R}^d)$ seen as an ordinary topological space. The most canonical topology to use is the strong topology.
When dealing with concrete models, it is a highly nontrivial task to construct this probability measure which a priori should be the weak limit of a sequence of well defined probability measures obtained by introducing ultraviolet and infrared cut-offs.






share|cite|improve this answer











$endgroup$













  • $begingroup$
    Thanks @AbdelmalekAbdesselam. Could you point out some reference showing how to rigorously define the measure (for instance in free theory) following this approach you mention of taking a weak limit of a sequence of measures obtained introducing UV and IR cut-offs? I believe that in free theory at least this is well understood, isn't it?
    $endgroup$
    – user1620696
    Dec 29 '18 at 18:21










  • $begingroup$
    @user1620696: In the free case you don't need to take weak limits of removing cutoffs. You can define the probability measure directly using the Bochner-Minlos Theorem. The most accessible reference I know is arxiv.org/abs/1706.09326 Note that pedagogically it also makes sense to define the free measure by taking a weak limit starting from a lattice with finite volume. The proof is easy using the Levy continuity theorem also proved in the reference I mentioned.
    $endgroup$
    – Abdelmalek Abdesselam
    Jan 2 at 11:32












  • $begingroup$
    Thanks for the reference. I think that will help. I'm also trying to connect with the usual (unrigorous) approach presented in most QFT books (like Peskin). I think the connection lies in this approach you mention of defining the free measure by taking a weak limit starting from a lattice with finite volume. At least this looks like what is done in these books, right?
    $endgroup$
    – user1620696
    Jan 2 at 11:45












  • $begingroup$
    I can't say since I don't have the S&P QFT book in front of me, but regardless of the choice of pedagogical introduction the Euclidean free measure obtained in the end is the one I hinted at via weak limits.
    $endgroup$
    – Abdelmalek Abdesselam
    Jan 2 at 11:59



















1












$begingroup$

Usually these are defined by a limiting process, splitting the QM path into lots of infinitesimal segments (or in Fourier space, by summing over a finite number of modes) and examining the continuous limit (if it exists). See for example the Feynman-Hibbs book. For field theory the path integral over field configurations is even more abstract and difficult to define.



If you don't like that, one may simply define the Gaussian functional integral in analogy to the discrete case (real variables and matrix in the exponent) and manipulate the path integral into moments of the Gaussian (essentially perturbation theory).



Incidentally, the worldline formalism to QFT replaces the functional field integral into a less abstract QM path integral.






share|cite|improve this answer









$endgroup$













    Your Answer





    StackExchange.ifUsing("editor", function () {
    return StackExchange.using("mathjaxEditing", function () {
    StackExchange.MarkdownEditor.creationCallbacks.add(function (editor, postfix) {
    StackExchange.mathjaxEditing.prepareWmdForMathJax(editor, postfix, [["$", "$"], ["\\(","\\)"]]);
    });
    });
    }, "mathjax-editing");

    StackExchange.ready(function() {
    var channelOptions = {
    tags: "".split(" "),
    id: "69"
    };
    initTagRenderer("".split(" "), "".split(" "), channelOptions);

    StackExchange.using("externalEditor", function() {
    // Have to fire editor after snippets, if snippets enabled
    if (StackExchange.settings.snippets.snippetsEnabled) {
    StackExchange.using("snippets", function() {
    createEditor();
    });
    }
    else {
    createEditor();
    }
    });

    function createEditor() {
    StackExchange.prepareEditor({
    heartbeatType: 'answer',
    autoActivateHeartbeat: false,
    convertImagesToLinks: true,
    noModals: true,
    showLowRepImageUploadWarning: true,
    reputationToPostImages: 10,
    bindNavPrevention: true,
    postfix: "",
    imageUploader: {
    brandingHtml: "Powered by u003ca class="icon-imgur-white" href="https://imgur.com/"u003eu003c/au003e",
    contentPolicyHtml: "User contributions licensed under u003ca href="https://creativecommons.org/licenses/by-sa/3.0/"u003ecc by-sa 3.0 with attribution requiredu003c/au003e u003ca href="https://stackoverflow.com/legal/content-policy"u003e(content policy)u003c/au003e",
    allowUrls: true
    },
    noCode: true, onDemand: true,
    discardSelector: ".discard-answer"
    ,immediatelyShowMarkdownHelp:true
    });


    }
    });














    draft saved

    draft discarded


















    StackExchange.ready(
    function () {
    StackExchange.openid.initPostLogin('.new-post-login', 'https%3a%2f%2fmath.stackexchange.com%2fquestions%2f2285761%2fis-there-some-approach-to-make-functional-integrals-rigorous%23new-answer', 'question_page');
    }
    );

    Post as a guest















    Required, but never shown

























    2 Answers
    2






    active

    oldest

    votes








    2 Answers
    2






    active

    oldest

    votes









    active

    oldest

    votes






    active

    oldest

    votes









    3












    $begingroup$

    There are two versions of such path integrals: the Minkowskian and the Euclidean.
    The first one is of the form
    $$
    int F(phi) e^{iS(phi)} Dphi
    $$

    and the second one is
    $$
    int F(phi) e^{-S(phi)} Dphi .
    $$

    Here $S$ is the action functional and $F$ is another functional corresponding to observables.
    The Minkowskian case lies outside ordinary measure theory, even in the Gaussian case (a Theorem by Cameron https://onlinelibrary.wiley.com/doi/abs/10.1002/sapm1960391126 correcting an earlier mistake by Gelfand and Yaglom who thought that $sigma$-additive complex measures would work). One instead has to use a limit of a time slicing procedure to make sense of it.



    For the Euclidean case, ordinary measure theory is perfectly adequate.
    The "I heard that as traditional measures it is not possible" is a common misconception that unfortunately gets repeated ad infinitum. The part $e^{-S(phi)} Dphi$ should be a Borel probability measure on a space of distributions like $mathscr{S}'(mathbb{R}^d)$ seen as an ordinary topological space. The most canonical topology to use is the strong topology.
    When dealing with concrete models, it is a highly nontrivial task to construct this probability measure which a priori should be the weak limit of a sequence of well defined probability measures obtained by introducing ultraviolet and infrared cut-offs.






    share|cite|improve this answer











    $endgroup$













    • $begingroup$
      Thanks @AbdelmalekAbdesselam. Could you point out some reference showing how to rigorously define the measure (for instance in free theory) following this approach you mention of taking a weak limit of a sequence of measures obtained introducing UV and IR cut-offs? I believe that in free theory at least this is well understood, isn't it?
      $endgroup$
      – user1620696
      Dec 29 '18 at 18:21










    • $begingroup$
      @user1620696: In the free case you don't need to take weak limits of removing cutoffs. You can define the probability measure directly using the Bochner-Minlos Theorem. The most accessible reference I know is arxiv.org/abs/1706.09326 Note that pedagogically it also makes sense to define the free measure by taking a weak limit starting from a lattice with finite volume. The proof is easy using the Levy continuity theorem also proved in the reference I mentioned.
      $endgroup$
      – Abdelmalek Abdesselam
      Jan 2 at 11:32












    • $begingroup$
      Thanks for the reference. I think that will help. I'm also trying to connect with the usual (unrigorous) approach presented in most QFT books (like Peskin). I think the connection lies in this approach you mention of defining the free measure by taking a weak limit starting from a lattice with finite volume. At least this looks like what is done in these books, right?
      $endgroup$
      – user1620696
      Jan 2 at 11:45












    • $begingroup$
      I can't say since I don't have the S&P QFT book in front of me, but regardless of the choice of pedagogical introduction the Euclidean free measure obtained in the end is the one I hinted at via weak limits.
      $endgroup$
      – Abdelmalek Abdesselam
      Jan 2 at 11:59
















    3












    $begingroup$

    There are two versions of such path integrals: the Minkowskian and the Euclidean.
    The first one is of the form
    $$
    int F(phi) e^{iS(phi)} Dphi
    $$

    and the second one is
    $$
    int F(phi) e^{-S(phi)} Dphi .
    $$

    Here $S$ is the action functional and $F$ is another functional corresponding to observables.
    The Minkowskian case lies outside ordinary measure theory, even in the Gaussian case (a Theorem by Cameron https://onlinelibrary.wiley.com/doi/abs/10.1002/sapm1960391126 correcting an earlier mistake by Gelfand and Yaglom who thought that $sigma$-additive complex measures would work). One instead has to use a limit of a time slicing procedure to make sense of it.



    For the Euclidean case, ordinary measure theory is perfectly adequate.
    The "I heard that as traditional measures it is not possible" is a common misconception that unfortunately gets repeated ad infinitum. The part $e^{-S(phi)} Dphi$ should be a Borel probability measure on a space of distributions like $mathscr{S}'(mathbb{R}^d)$ seen as an ordinary topological space. The most canonical topology to use is the strong topology.
    When dealing with concrete models, it is a highly nontrivial task to construct this probability measure which a priori should be the weak limit of a sequence of well defined probability measures obtained by introducing ultraviolet and infrared cut-offs.






    share|cite|improve this answer











    $endgroup$













    • $begingroup$
      Thanks @AbdelmalekAbdesselam. Could you point out some reference showing how to rigorously define the measure (for instance in free theory) following this approach you mention of taking a weak limit of a sequence of measures obtained introducing UV and IR cut-offs? I believe that in free theory at least this is well understood, isn't it?
      $endgroup$
      – user1620696
      Dec 29 '18 at 18:21










    • $begingroup$
      @user1620696: In the free case you don't need to take weak limits of removing cutoffs. You can define the probability measure directly using the Bochner-Minlos Theorem. The most accessible reference I know is arxiv.org/abs/1706.09326 Note that pedagogically it also makes sense to define the free measure by taking a weak limit starting from a lattice with finite volume. The proof is easy using the Levy continuity theorem also proved in the reference I mentioned.
      $endgroup$
      – Abdelmalek Abdesselam
      Jan 2 at 11:32












    • $begingroup$
      Thanks for the reference. I think that will help. I'm also trying to connect with the usual (unrigorous) approach presented in most QFT books (like Peskin). I think the connection lies in this approach you mention of defining the free measure by taking a weak limit starting from a lattice with finite volume. At least this looks like what is done in these books, right?
      $endgroup$
      – user1620696
      Jan 2 at 11:45












    • $begingroup$
      I can't say since I don't have the S&P QFT book in front of me, but regardless of the choice of pedagogical introduction the Euclidean free measure obtained in the end is the one I hinted at via weak limits.
      $endgroup$
      – Abdelmalek Abdesselam
      Jan 2 at 11:59














    3












    3








    3





    $begingroup$

    There are two versions of such path integrals: the Minkowskian and the Euclidean.
    The first one is of the form
    $$
    int F(phi) e^{iS(phi)} Dphi
    $$

    and the second one is
    $$
    int F(phi) e^{-S(phi)} Dphi .
    $$

    Here $S$ is the action functional and $F$ is another functional corresponding to observables.
    The Minkowskian case lies outside ordinary measure theory, even in the Gaussian case (a Theorem by Cameron https://onlinelibrary.wiley.com/doi/abs/10.1002/sapm1960391126 correcting an earlier mistake by Gelfand and Yaglom who thought that $sigma$-additive complex measures would work). One instead has to use a limit of a time slicing procedure to make sense of it.



    For the Euclidean case, ordinary measure theory is perfectly adequate.
    The "I heard that as traditional measures it is not possible" is a common misconception that unfortunately gets repeated ad infinitum. The part $e^{-S(phi)} Dphi$ should be a Borel probability measure on a space of distributions like $mathscr{S}'(mathbb{R}^d)$ seen as an ordinary topological space. The most canonical topology to use is the strong topology.
    When dealing with concrete models, it is a highly nontrivial task to construct this probability measure which a priori should be the weak limit of a sequence of well defined probability measures obtained by introducing ultraviolet and infrared cut-offs.






    share|cite|improve this answer











    $endgroup$



    There are two versions of such path integrals: the Minkowskian and the Euclidean.
    The first one is of the form
    $$
    int F(phi) e^{iS(phi)} Dphi
    $$

    and the second one is
    $$
    int F(phi) e^{-S(phi)} Dphi .
    $$

    Here $S$ is the action functional and $F$ is another functional corresponding to observables.
    The Minkowskian case lies outside ordinary measure theory, even in the Gaussian case (a Theorem by Cameron https://onlinelibrary.wiley.com/doi/abs/10.1002/sapm1960391126 correcting an earlier mistake by Gelfand and Yaglom who thought that $sigma$-additive complex measures would work). One instead has to use a limit of a time slicing procedure to make sense of it.



    For the Euclidean case, ordinary measure theory is perfectly adequate.
    The "I heard that as traditional measures it is not possible" is a common misconception that unfortunately gets repeated ad infinitum. The part $e^{-S(phi)} Dphi$ should be a Borel probability measure on a space of distributions like $mathscr{S}'(mathbb{R}^d)$ seen as an ordinary topological space. The most canonical topology to use is the strong topology.
    When dealing with concrete models, it is a highly nontrivial task to construct this probability measure which a priori should be the weak limit of a sequence of well defined probability measures obtained by introducing ultraviolet and infrared cut-offs.







    share|cite|improve this answer














    share|cite|improve this answer



    share|cite|improve this answer








    edited Jan 2 at 11:26

























    answered Aug 9 '18 at 15:55









    Abdelmalek AbdesselamAbdelmalek Abdesselam

    667311




    667311












    • $begingroup$
      Thanks @AbdelmalekAbdesselam. Could you point out some reference showing how to rigorously define the measure (for instance in free theory) following this approach you mention of taking a weak limit of a sequence of measures obtained introducing UV and IR cut-offs? I believe that in free theory at least this is well understood, isn't it?
      $endgroup$
      – user1620696
      Dec 29 '18 at 18:21










    • $begingroup$
      @user1620696: In the free case you don't need to take weak limits of removing cutoffs. You can define the probability measure directly using the Bochner-Minlos Theorem. The most accessible reference I know is arxiv.org/abs/1706.09326 Note that pedagogically it also makes sense to define the free measure by taking a weak limit starting from a lattice with finite volume. The proof is easy using the Levy continuity theorem also proved in the reference I mentioned.
      $endgroup$
      – Abdelmalek Abdesselam
      Jan 2 at 11:32












    • $begingroup$
      Thanks for the reference. I think that will help. I'm also trying to connect with the usual (unrigorous) approach presented in most QFT books (like Peskin). I think the connection lies in this approach you mention of defining the free measure by taking a weak limit starting from a lattice with finite volume. At least this looks like what is done in these books, right?
      $endgroup$
      – user1620696
      Jan 2 at 11:45












    • $begingroup$
      I can't say since I don't have the S&P QFT book in front of me, but regardless of the choice of pedagogical introduction the Euclidean free measure obtained in the end is the one I hinted at via weak limits.
      $endgroup$
      – Abdelmalek Abdesselam
      Jan 2 at 11:59


















    • $begingroup$
      Thanks @AbdelmalekAbdesselam. Could you point out some reference showing how to rigorously define the measure (for instance in free theory) following this approach you mention of taking a weak limit of a sequence of measures obtained introducing UV and IR cut-offs? I believe that in free theory at least this is well understood, isn't it?
      $endgroup$
      – user1620696
      Dec 29 '18 at 18:21










    • $begingroup$
      @user1620696: In the free case you don't need to take weak limits of removing cutoffs. You can define the probability measure directly using the Bochner-Minlos Theorem. The most accessible reference I know is arxiv.org/abs/1706.09326 Note that pedagogically it also makes sense to define the free measure by taking a weak limit starting from a lattice with finite volume. The proof is easy using the Levy continuity theorem also proved in the reference I mentioned.
      $endgroup$
      – Abdelmalek Abdesselam
      Jan 2 at 11:32












    • $begingroup$
      Thanks for the reference. I think that will help. I'm also trying to connect with the usual (unrigorous) approach presented in most QFT books (like Peskin). I think the connection lies in this approach you mention of defining the free measure by taking a weak limit starting from a lattice with finite volume. At least this looks like what is done in these books, right?
      $endgroup$
      – user1620696
      Jan 2 at 11:45












    • $begingroup$
      I can't say since I don't have the S&P QFT book in front of me, but regardless of the choice of pedagogical introduction the Euclidean free measure obtained in the end is the one I hinted at via weak limits.
      $endgroup$
      – Abdelmalek Abdesselam
      Jan 2 at 11:59
















    $begingroup$
    Thanks @AbdelmalekAbdesselam. Could you point out some reference showing how to rigorously define the measure (for instance in free theory) following this approach you mention of taking a weak limit of a sequence of measures obtained introducing UV and IR cut-offs? I believe that in free theory at least this is well understood, isn't it?
    $endgroup$
    – user1620696
    Dec 29 '18 at 18:21




    $begingroup$
    Thanks @AbdelmalekAbdesselam. Could you point out some reference showing how to rigorously define the measure (for instance in free theory) following this approach you mention of taking a weak limit of a sequence of measures obtained introducing UV and IR cut-offs? I believe that in free theory at least this is well understood, isn't it?
    $endgroup$
    – user1620696
    Dec 29 '18 at 18:21












    $begingroup$
    @user1620696: In the free case you don't need to take weak limits of removing cutoffs. You can define the probability measure directly using the Bochner-Minlos Theorem. The most accessible reference I know is arxiv.org/abs/1706.09326 Note that pedagogically it also makes sense to define the free measure by taking a weak limit starting from a lattice with finite volume. The proof is easy using the Levy continuity theorem also proved in the reference I mentioned.
    $endgroup$
    – Abdelmalek Abdesselam
    Jan 2 at 11:32






    $begingroup$
    @user1620696: In the free case you don't need to take weak limits of removing cutoffs. You can define the probability measure directly using the Bochner-Minlos Theorem. The most accessible reference I know is arxiv.org/abs/1706.09326 Note that pedagogically it also makes sense to define the free measure by taking a weak limit starting from a lattice with finite volume. The proof is easy using the Levy continuity theorem also proved in the reference I mentioned.
    $endgroup$
    – Abdelmalek Abdesselam
    Jan 2 at 11:32














    $begingroup$
    Thanks for the reference. I think that will help. I'm also trying to connect with the usual (unrigorous) approach presented in most QFT books (like Peskin). I think the connection lies in this approach you mention of defining the free measure by taking a weak limit starting from a lattice with finite volume. At least this looks like what is done in these books, right?
    $endgroup$
    – user1620696
    Jan 2 at 11:45






    $begingroup$
    Thanks for the reference. I think that will help. I'm also trying to connect with the usual (unrigorous) approach presented in most QFT books (like Peskin). I think the connection lies in this approach you mention of defining the free measure by taking a weak limit starting from a lattice with finite volume. At least this looks like what is done in these books, right?
    $endgroup$
    – user1620696
    Jan 2 at 11:45














    $begingroup$
    I can't say since I don't have the S&P QFT book in front of me, but regardless of the choice of pedagogical introduction the Euclidean free measure obtained in the end is the one I hinted at via weak limits.
    $endgroup$
    – Abdelmalek Abdesselam
    Jan 2 at 11:59




    $begingroup$
    I can't say since I don't have the S&P QFT book in front of me, but regardless of the choice of pedagogical introduction the Euclidean free measure obtained in the end is the one I hinted at via weak limits.
    $endgroup$
    – Abdelmalek Abdesselam
    Jan 2 at 11:59











    1












    $begingroup$

    Usually these are defined by a limiting process, splitting the QM path into lots of infinitesimal segments (or in Fourier space, by summing over a finite number of modes) and examining the continuous limit (if it exists). See for example the Feynman-Hibbs book. For field theory the path integral over field configurations is even more abstract and difficult to define.



    If you don't like that, one may simply define the Gaussian functional integral in analogy to the discrete case (real variables and matrix in the exponent) and manipulate the path integral into moments of the Gaussian (essentially perturbation theory).



    Incidentally, the worldline formalism to QFT replaces the functional field integral into a less abstract QM path integral.






    share|cite|improve this answer









    $endgroup$


















      1












      $begingroup$

      Usually these are defined by a limiting process, splitting the QM path into lots of infinitesimal segments (or in Fourier space, by summing over a finite number of modes) and examining the continuous limit (if it exists). See for example the Feynman-Hibbs book. For field theory the path integral over field configurations is even more abstract and difficult to define.



      If you don't like that, one may simply define the Gaussian functional integral in analogy to the discrete case (real variables and matrix in the exponent) and manipulate the path integral into moments of the Gaussian (essentially perturbation theory).



      Incidentally, the worldline formalism to QFT replaces the functional field integral into a less abstract QM path integral.






      share|cite|improve this answer









      $endgroup$
















        1












        1








        1





        $begingroup$

        Usually these are defined by a limiting process, splitting the QM path into lots of infinitesimal segments (or in Fourier space, by summing over a finite number of modes) and examining the continuous limit (if it exists). See for example the Feynman-Hibbs book. For field theory the path integral over field configurations is even more abstract and difficult to define.



        If you don't like that, one may simply define the Gaussian functional integral in analogy to the discrete case (real variables and matrix in the exponent) and manipulate the path integral into moments of the Gaussian (essentially perturbation theory).



        Incidentally, the worldline formalism to QFT replaces the functional field integral into a less abstract QM path integral.






        share|cite|improve this answer









        $endgroup$



        Usually these are defined by a limiting process, splitting the QM path into lots of infinitesimal segments (or in Fourier space, by summing over a finite number of modes) and examining the continuous limit (if it exists). See for example the Feynman-Hibbs book. For field theory the path integral over field configurations is even more abstract and difficult to define.



        If you don't like that, one may simply define the Gaussian functional integral in analogy to the discrete case (real variables and matrix in the exponent) and manipulate the path integral into moments of the Gaussian (essentially perturbation theory).



        Incidentally, the worldline formalism to QFT replaces the functional field integral into a less abstract QM path integral.







        share|cite|improve this answer












        share|cite|improve this answer



        share|cite|improve this answer










        answered May 18 '17 at 0:23









        luxlux

        1637




        1637






























            draft saved

            draft discarded




















































            Thanks for contributing an answer to Mathematics Stack Exchange!


            • Please be sure to answer the question. Provide details and share your research!

            But avoid



            • Asking for help, clarification, or responding to other answers.

            • Making statements based on opinion; back them up with references or personal experience.


            Use MathJax to format equations. MathJax reference.


            To learn more, see our tips on writing great answers.




            draft saved


            draft discarded














            StackExchange.ready(
            function () {
            StackExchange.openid.initPostLogin('.new-post-login', 'https%3a%2f%2fmath.stackexchange.com%2fquestions%2f2285761%2fis-there-some-approach-to-make-functional-integrals-rigorous%23new-answer', 'question_page');
            }
            );

            Post as a guest















            Required, but never shown





















































            Required, but never shown














            Required, but never shown












            Required, but never shown







            Required, but never shown

































            Required, but never shown














            Required, but never shown












            Required, but never shown







            Required, but never shown







            Popular posts from this blog

            Quarter-circle Tiles

            build a pushdown automaton that recognizes the reverse language of a given pushdown automaton?

            Mont Emei